2025 AMC 10A Problems/Problem 17
(Problem goes here)
Problem
Let
be the unique positive integer such that dividing
by
leaves a remainder of
and dividing
by
leaves a remainder of
. What is the tens digit of
?
Solution 1
The problem statement implies
and
We want to find
that satisfies both of these conditions. Hence, we can just find the greatest common divisor of the two numbers.
Which simplifies to
by the Euclidean Algorithm so the answer is
~Tacos_are_yummy_1
Solution 2
We have: 273436 ≡ 16 (mod N) 272760 ≡ 15 (mod N)
First we substract (273436 − 272760) = 676 ≡ 1 (mod N)
So N divides 675. Since N is greater than 16, possible divisors are all greater than 16 and are 25, 27, 45, 75, 135, 225, 675. We then check which ones work. If 273436 ≡ 16 (mod N), then 273420 must be divisible by N. 273420 ÷ 45 = 6076, so N = 45 works. So N = 45, and the tens digit is
.
~Continuous_Pi
Alternate Solution
We are given \(273436 \equiv 16 \pmod{N}\) and \(272760 \equiv 15 \pmod{N}.\) Subtracting the second equation from the first yields \(273436 - 272760 \equiv 16 - 15 \pmod{N}.\) so now, \(676 \equiv 1 \pmod{N}.\) Hence, \(N \mid (676 - 1) = 675.\) The prime factorization of 675 is \(675 = 5^2 \times 3^3.\) Therefore, \(N\) must be a divisor of 675. Since the remainders are less than \(N,\) we have \(N > 16.\) The largest divisor of 675 greater than 16 is \(45.\) The tens digit of 45 is 4. Thus, the answer is \(\boxed{\text{(E) }4}.\)
~WildSealVM/Vincent M.